LSAT and Law School Admissions Forum

Get expert LSAT preparation and law school admissions advice from PowerScore Test Preparation.

 Administrator
PowerScore Staff
  • PowerScore Staff
  • Posts: 8916
  • Joined: Feb 02, 2011
|
#41383
Please post your questions below!
 g_lawyered
  • Posts: 211
  • Joined: Sep 14, 2020
|
#92860
Hi P.S.
I translated the question stem to mean we're looking for the correct answer to be a Must be False (the 4 incorrect answers are: Could be True). Just want to make sure than when translating the Logical Opposite with EXCEPT that:

CBT EXCEPT- the correct answer is Must be False
CBF EXCEPT- the correct answer is Must be True

^^ Is this correct?

I was able to narrow down contenders to A and B. Because I was pressed on time I drew out hypothetical choice A to see if it was a Must be False and didn't test out answer choice B (I picked A once I saw it was a Must be False & eliminated B). There are 2 hypotheticals I drew to test answer choice A (and determined it was a Must be False answer- correct answer).
(I drew hypothetical 1 out during PT and hypothetical 2 once I reviewed this game- as I was rushing throughout my PT).

Hypothetical 1: Answer choice A (G in 3rd)

(In order 1-3)
W: M, __ , __
E: ___, U, G

In this hypothetical, per rule 1, S must be in West so it must go in either 2 or 3. and O and P must go either 2 or 3 (since M is 1st). This would force either O or P to be 1st in East (which breaks rule 3- that M must go BEFORE O and P. O and P CAN'T go at the same time as M meaning neither can go 1st).

Hypothetical 2: that proves A is a Must be False is (G in 3rd):
(In order 1-3)
W: M, P/O, G
E: __, U, P/O

This hypothetical has the same 2 problems I mentioned hypothetical 1 has. It would break per rule 1 and rule 3.

Can someone show me a hypothetical that proves that answer choice A is incorrect? That it Could be true??

Now that I reviewed my PT and am not under time, I was able to draw out Answer choice B and see why B is a Must be False answer (the correct answer). Now, I just can't wrap my head around why answer choice A isn't a Must be False answer. :-?

Can someone please demonstrate to help clarify. Thanks in advance!
 Robert Carroll
PowerScore Staff
  • PowerScore Staff
  • Posts: 1787
  • Joined: Dec 06, 2013
|
#92876
GGIBA003,

Yes, Could Be True EXCEPT = one Cannot Be True/Must Be False

and

Could Be False EXCEPT = one Must Be True

By the time you get to questions #5, you've very likely done the other questions and have some work that can help you answer this one without having to waste time repeating a lot of work. For instance, for question #2, you could have the following hypothetical:

West: SOG
East MUP

In fact, I do have that, based on my answer in this thread: viewtopic.php?p=92874#p92874

So, when U is at 2, we know all of the following could be true:

G can be at 3 (answer choice (A))
O can be at 2 (answer choice (C))
S can be at 1 (answer choice (E))

Only two answer choices are left, and you can brute-force one of them. If it works, it's wrong, and you pick the other answer. If it doesn't work, you pick it. Either way, one more diagram answers this.

Your first hypothetical for answer choice (A) is only one possibility among many - you're putting U, G, and M into specific theaters. You're showing that that arrangement doesn't work. That only proves that, if you put those three variables are certain times AND certain theaters, it won't work. Why not change the theaters a bit? In fact, as my hypothetical from question #2 shows, we can make answer choice (A) work, but it requires the theaters to be different than what your hypothetical showed. That's fine, because nothing about the local condition for question #5, nor any of the answer choices, determines which theaters things are in. So, ultimately, you're showing that in ONE hypothetical compatible with answer choice (A), the rules are violated. In order to show that answer choice (A) cannot be true at all, you'd have to show that ALL hypotheticals compatible with answer choice (A) violate some rule. Basically, you proved that answer choice (A) doesn't have to be true. You needed to go further - prove it can't be true. Since it can be true, that wouldn't have been possible.

Robert Carroll
 g_lawyered
  • Posts: 211
  • Joined: Sep 14, 2020
|
#92924
During my PT, I did question 5 but skipped and guessed on question 2, which is why I think I wasn't able to use previous work to see that answer choice is a CBT not a MBF. Since I was rushing, I didn't have time to draw out a different template for answer choice A so I just picked it. Were there or did we need templates in this game that could've helped answer questions faster? I didn't draw out any templates during my PT...
Thanks for clarifying that up.
 Rachael Wilkenfeld
PowerScore Staff
  • PowerScore Staff
  • Posts: 1358
  • Joined: Dec 15, 2011
|
#92941
Hi GGIBA,

I think this one is doable without templates or prior work. Prior work is useful on these types of questions because it can help eliminate options that can be true.

Here I'd start by setting up the scenario

East 1: __ 2: U 3: __
West 1:__2:__ 3___

Once U is at 2, we know G has to be one of our two 3 slots.

At this point, I'd look at my diagram or rules. The main sequencing rule left is that M has to be before both O and P. Now we are talking! Normally, M could go at either 1 or 2. But here, if we put M at 2, we'd need G, O and P at 3pm. That's too many at 3pm. We would violate our rules. Therefore, we know M can't be in 2. That's answer choice (B). We don't have to check the other choices since we can see that it violates the rules.

Hope that helps!
 g_lawyered
  • Posts: 211
  • Joined: Sep 14, 2020
|
#93441
I just wanted to know if I had missed a key element in this game that could've allowed me to go through the questions faster. But I see that i didn't it. Thank you Kelly

Get the most out of your LSAT Prep Plus subscription.

Analyze and track your performance with our Testing and Analytics Package.